From: W. Trevor King Date: Wed, 11 Apr 2012 17:28:23 +0000 (-0400) Subject: Fix problem* vs. problem62 in Serway and Jewett v4's problem 1.62. X-Git-Url: http://git.tremily.us/?p=course.git;a=commitdiff_plain;h=a7323c0c4ccaeb9e432130c485833aacefbb52bd Fix problem* vs. problem62 in Serway and Jewett v4's problem 1.62. --- diff --git a/latex/problems/Serway_and_Jewett_4/problem01.62.tex b/latex/problems/Serway_and_Jewett_4/problem01.62.tex index 447af74..c81831c 100644 --- a/latex/problems/Serway_and_Jewett_4/problem01.62.tex +++ b/latex/problems/Serway_and_Jewett_4/problem01.62.tex @@ -7,7 +7,7 @@ Demonstrate that for small angles ($< 20\dg$) where $\alpha$ is in radians and $\alpha '$ is in degrees. Use a calculator to find the largest angle for which $\tan \alpha$ may be approximated by $\alpha$ with an error less than $10.0$\%. -\end{problem62} % Problem 1.62 +\end{problem*} \begin{solution} To kill both birds with one stone,